Solve for the total cost. Retail price: $12.95; discount rate: 15%; sales tax rate: 7%

Answers

Answer 1

Answer: $10.24

Step-by-step explanation: 12.95 * 0.15 = 1.94    12.95 - 1.94 = 11.01

11.01 * 0.07 = .77      11.01 - 0.77 = 10.24


Related Questions

A hat contains four slips of paper. The slips of paper are numbered 3, 3, 6, and 8. One slip of paper is randomly selected and not replaced, and then a second slip of paper is selected. the numbers on the two slips of paper are added together.

A fair decision is to be made about which one of three sizes of coffee will be ordered, using the sum of the numbers on the slips of paper.

The coffee size options are small, medium, and large.

Which description accurately explains how a fair decision can be made in this situation?

A.) If the sum of the slips of paper is 9, a small coffee will be ordered. if the sum of the slips of paper is 11, a medium coffee will be ordered. If the sum of the slips of paper is 6 or 14, a large coffee will be ordered.

B.) If the sum of the slips of paper is 9, a small coffee will be ordered. if the sum of the slips of paper is 6, a medium coffee will be ordered. If the sum of the slips of paper is 11 or 14, a large coffee will be ordered.

C.) If the sum of the slips of paper is 9, a small coffee will be ordered. if the sum of the slips of paper is 14, a medium coffee will be ordered. If the sum of the slips of paper is 6 or 11, a large coffee will be ordered.

D.) If the sum of the slips of paper is 14, a small coffee will be ordered. if the sum of the slips of paper is 9, a medium coffee will be ordered. If the sum of the slips of paper is 6 or 11, a large coffee will be ordered.

Answers

The option that  accurately  fit the description that  explains how a fair decision can be made in this situation is option A - the sum of the slips of paper is 9.

What is Random selection?

This is known to be  scientific method used when each unit in a population is known or said to have an equal chance of been selection for  any inclusion into a sample.

The option that  accurately  fit the description that  explains how a fair decision can be made in this situation is option A - the sum of the slips of paper is 9, a small coffee will be ordered. if the sum of the slips of paper is 11, a medium coffee will be ordered. If the sum of the slips of paper is 6 or 14, a large coffee will be ordered.

Learn more about random selection from

https://brainly.com/question/24466382

#SPJ1

What is the solution to the equation?
5 = 2/5 a

O 2

O 4 3/5

O 12 1/2

O 25

Answers

Answer:

Option C, [tex]12\frac{1}{2}[/tex]

Step-by-step explanation:

Step 1:  Multiply both sides by 5/2

[tex]5 = \frac{2}{5}a[/tex]

[tex]5 * \frac{5}{2} = \frac{2}{5}a * \frac{5}{2}[/tex]

[tex]\frac{25}{2}=a[/tex]

[tex]12\frac{1}{2}=a[/tex]

Answer: Option C, [tex]12\frac{1}{2}[/tex]

Hope this helps!  Let me know if you have any questions

What is the exact value of tan 75?

Answers

Answer:

2 + √3.

Step-by-step explanation:

tan 75 = tan (45 + 30)

           =     (tan 45 + tan 30)  /  (1 - tan 45 tan 30)

           =   ( 1 + 1/√3) / (1 - 1*1/√3)

           =   (1 + √3) / √3  *   √3 / (√3 - 1)

           = √3 + 1/ √3 - 1

  Multiply top and bottom by √3 + 1;

                =  (√3 + 1)(√3 + 1) /  (√3 -  1)(√3 + 1)

                = 3 + 2√3 + 1 / (3 - 1)

                =  (4 + 2√3) / 2

                =  2 + √3.

               

What is the value represented by the letter E on the box plot of the data?
45, 45, 50, 55, 60, 65, 72, 75, 80, 80, 90
А
B.
с
D
E
Enter your answer in the box.

Answers

Answer:

E = 90

Step-by-step explanation:

E represents the maximum value in the data set , that is

E represents the value 90

Answer:

E is 90

Step-by-step explanation:

the end of the box and whisker plot is the last number in chronoligical order. So the one all the way on the right, e is 90

a photograph measuring 8 cm by 10cm costs 440 naira. what will be the cost of an enlargement measuring 20cm by 25cm

Answers

Answer:

2750

Step-by-step explanation:

Remark

You have to assume that this can be solved by using a proportion. A proportion is 2 equal ratios. Three parts of the 2 ratios must be known and your job is to find the 4th one.

Equation

w*L/C1 = w1*L1 / x

Solution

w = 8

L = 10

w1 = 20

L1  = 25

C1 = 449

C2= x

Solution

8*10/440 = 20*25/x          Combine

80/440 = 500 / x               Cross Multiply

80x = 440*500                  Combine

80x = 220000                  Divide by 80

x = 220000/80

Answer

x = 2750

helppp asapppppppppp

Answers

Answer:

put a point on the 9 (the line on the left of 10) and on the -6 (the line to the left of -5).

a. 9 is located to the right of -6

b. use the symbol >

Step-by-step explanation:

PLEASE HELP ME IM HONESTY SO CONFUSED WITH THE ANSWERS GIVEN

Answers

Answer:

C. The sum of 18 and half the product of 9 and 4.

good evening! Can someone please answer this, ill give you brainliest and your earning 50 points. Would be very appreciated.

Answers

Answer:

Part 1

Set B

Part 2

We can use the FOIL method to multiply these expressions:

[tex]\textsf{FOIL}: \quad(a+b)(c+d)=ac+ad+bc+bd[/tex]

However, as all the expression are in the format [tex](a+b)^2[/tex], we can use the shortcut:  [tex](a+b)^2=a^2+2ab+b^2[/tex]

Part 3

Using the shortcut to find the products:

[tex]\begin{aligned}\implies (x+5)(x+5)& =(x+5)^2\\ & =x^2+2(x)(5)+5^2\\ & =x^2+10x+25\end{aligned}[/tex]

[tex]\begin{aligned}\implies (2x+9)(2x+9) & = (2x+9)^2\\ & = (2x)^2+2(2x)(9)+9^2\\ & = 4x^2+36x+81\end{aligned}[/tex]

[tex]\begin{aligned}\implies (x+1)^2 & = x^2+2(x)(1)+1^2\\ & = x^2+2x+1\end{aligned}[/tex]

Part 4

A new example of a multiplication problem based on the structure of Set B is:  [tex](3x+2)(3x+2)[/tex]

[tex]\begin{aligned}\implies (3x+2)(3x+2) & = (3x+2)^2\\ & = (3x)^2+2(3x)(2)+2^2\\ & = 9x^2+12x+4\end{aligned}[/tex]

Part 5

An example of a multiplication problem that would NOT be included in Set B based on its structure is: [tex](x+2)(x-2)[/tex]

We cannot use the derived Set B shortcut (from part 2) to multiply this expression.  Instead, we would have to use a different shortcut of "The Difference of Two Squares" to find the product of this example expression.

Please answer these questions for me

Answers

Answer:

Do reciprocal of that and diameter formula and when your answer will come me just do square multiple of that

Factor completely x2 - 6x2 – 5x + 30.
(x + 6)(x2 + 5)
(x - 6)(x2 + 5)
(x - 6)(x2 – 5)
(x + 1)(x2 - 5)

Answers

Step-by-step explanation:

−5x2−5x+30

=5(−x+2)(x+3)

Answer:

5(−x+2)(x+3)

Find the missing value so
that the two points have a
slope of 2/7.
(-1,-1) and (x,1)

Answers

Answer:

x = 6

Step-by-step explanation:

Slope = (y(2) - y(1))/(x(2) - x(1))

(-1 - 1) / (-1 - x)

If the slope is 2/7, it would be (-2/-1-x, where x would have to be 6)

(-2)/(-7) = 2/7, so x = 6

Check:

(-1 - 1) / (-1 - 6)

(-2)/(-7)

2/7 (Correct :))

Hope it helps :D

PLEASE WILL GIVE 50 POINTS PLEASE ANSWER ​

Answers

Answer:

AB = 75

BC = 60

AC = 45

m∠A = 53°

m∠B = 37°

m∠C = 90°

Step-by-step explanation:

Trigonometric ratios

[tex]\sf \sin(\theta)=\dfrac{O}{H}\quad\cos(\theta)=\dfrac{A}{H}\quad\tan(\theta)=\dfrac{O}{A}[/tex]

where:

[tex]\theta[/tex] is the angleO is the side opposite the angleA is the side adjacent the angleH is the hypotenuse (the side opposite the right angle)

Given:

[tex]\sf \tan(A)=\dfrac{60}{45}[/tex]

Therefore:

side opposite angle A = BC = 60side adjacent angle A = AC = 45

To find the length of AB (the hypotenuse), use Pythagoras’ Theorem:

[tex]a^2+b^2=c^2[/tex]

(where a and b are the legs, and c is the hypotenuse, of a right triangle)

⇒ AC² + BC² = AB²

⇒ 45² + 60² = AB²

⇒ AB² = 5625

⇒ AB = √5625

AB = 75

To find m∠A:

[tex]\implies\sf \tan(A)=\dfrac{60}{45}[/tex]

[tex]\implies\sf A=\tan^{-1}\left(\dfrac{60}{45}\right)[/tex]

[tex]\implies\sf A=53^{\circ}\:(nearest\:degree)[/tex]

m∠C = 90° (as it is a right angle)

The interior angles of a triangle sum to 180°

⇒ m∠A + m∠B + m∠C = 180°

⇒ 53° + m∠B + 90° = 180°

⇒ m∠B = 180° - 53° - 90°

m∠B = 37°

À point has been translated left and down. Based on the graph, which could be true? Check all that apply.

Answers

Answer:

Step-by-step explanation:

B is the image of D

6 left

2 down

Answer: C (in choices)

B is the image E

6 down

5 left

Answer: D  (in choices)

C is the image of D

1 left

2 down

Answer: F (in choices)

Which choice is equal to 66
6
6
+ 66
6
6
+ 66
6
6
+ 36
3
6
+ 26
2
6
?

Answers

There are 11 1/8 picec that equals 1 3/8

help me please help ​

Answers

Step-by-step explanation:

n + 34 = 34 + n

commutative means you can change the sequence of the parts without changing the overall result.

g × h = h × g

s + 0 = s

b × 1 = b

the identity property points are to show that adding 0 or multiplying by 1 does not change anything.

Find each value or measure. Assume that segments that appear to be tangent are tangent.

Answers

Answer:

Step-by-step explanation:

13). ( x - 2 )( x + 6 ) = 9 × 20

x² + 4x - 12 = 180

x² + 4x - 192 = 0

( x - 12 )( x + 16 ) = 0

[tex]x_{1}[/tex] = - 16

[tex]x_{2}[/tex] = 12

14). ( x + 4 )[( x + 4 ) + 3x ] = 8 × ( 8 + 19 )

( x + 4 )( 4x + 4 ) = 216

4( x + 4 )( x + 1 ) = 216

( x + 4 )( x + 1 ) = 54

x² + 5x + 4 - 54 = 0

x² + 5x - 50 = 0

( x - 5 )( x + 10 ) = 0

[tex]x_{1}[/tex] = - 10

[tex]x_{2}[/tex] = 5

15). x( x + 16 ) = 15²

x² + 16x - 225 = 0

( x - 9 )( x + 25 ) = 0

[tex]x_{1}[/tex] = - 25

[tex]x_{2}[/tex] = 9

16). Now is your turn. You can do it!

The measurements are given in the solution below.

What is a circle?

Circle definition states a shape that consists of points, in a two-dimensional plane, equidistant from a given point.

Given that the circles we need to find the given missing values,

Using the properties of the circles,

1) (x-2) (x+6) = 20 × 9

x² + 6x - 2x - 12 = 180

x² + 4x - 192 = 0

x = 12 or x = -16.

2) 8 × (8+19) = x+4 (x+4 + 3x)

216 = x + 4 × (4x + 4)

216 = 4x² + 4x + 16x + 16

4x² + 20x - 200 = 0

x² + 5x - 50 = 0

x = 5 or x = -10

3) 15² = 16x

x = 225/16

x = 14.06
4) (x+7)² = 18 × 14

x² + 49 + 14x = 252

x² + 14x - 203 = 0

x = 8.87

x = -22.87

Learn more about circles click;

https://brainly.com/question/29142813

#SPJ2

what is a solution of √7-2x+5=8

Answers

Answer: √7 / 2 - 3/2

Step-by-step explanation:

√7-2x+5=8
=>  -2x + √7 = 3
=> x =  √7 / 2 - 3/2

PLEASE ANSWER AS SOON AS U Can

Answers

Answer:  3082.2

Step-by-step explanation:

Answer: 3,082.2 ft

Step-by-step explanation:

You must apply the formula for calculate the perimeter of a circle, which is:

P=πd where,

P is the perimeter of the circle. (P=9,687 ft)

d is the diameter of the circle.

When you solve for d, you get:

d = P/π

d = (9,687 ft)/(22/7)

d = 3,082.2 ft

Therefore, the answer is: 3,082.2 ft

Find the value of the variable x (2x+12) and 8x-18

Answers

Answer: Below

Step-by-step explanation:

Just mutiply x on both sides which gives you 2x^2 + 12x then you mutiply 2x^2 then subtract that number on both sides and it gives you x = something then you distribute it on 8(something) - 18 you subtract

Answer:

Step-by-step explanation:

Just mutiply x on both sides which gives you 2x^2 + 12x then you mutiply 2x^2 then subtract that number on both sides and it gives you x = something then you distribute it on 8(something) - 18 you subtract

The functions f(x) and g(x) are described using the following equation and table:

f(x) = −3(1.02)x

x g(x)
−1 −5
0 −3
1 −1
2 1
Which statement best compares the y-intercepts of f(x) and g(x)? (5 points)

Group of answer choices

The y-intercept of f(x) is equal to the y-intercept of g(x).

The y-intercept of f(x) is equal to 2 times the y-intercept of g(x).

The y-intercept of g(x) is equal to 2 times the y-intercept of f(x).

The y-intercept of g(x) is equal to 2 plus the y-intercept of f(x).

Answers

Answer:

A) The y-intercept of f(x) is equal to the y-intercept of g(x).

Step-by-step explanation:

Given functions f(x) and g(x).

What is the y-intercept?

The y-intercept is the output of the function with the input of zero.

In other words the y -intercept is:

f(0) for f(x) and g(0) for g(x)

Let's determine the y-intercept of the given functions:

f(0) = - 3(1.02)⁰ = - 3(1) = - 3

g(0) = - 3 according to table

We see both functions have same y-intercept with the value of -3.

The matching answer choice is A.

A spinner is divided into 4 equal sectors that are colored green, red, yellow, and blue. The number of times the spinner lands on each color is listed below. image Which statement best explains whether the results listed in the table should cause the probability model for the given situation to be questioned?

Answers

The probability of landing on the 4 spinners divided into four equal parts will be P=25%

What is probability?

Probability is defined as the ratio of the number of favorable outcomes to the total number of outcomes in other words the probability is the number that shows the happening of the event.

Here the spinner is divided into four parts of the colours green, red, yellow, and blue.So the number of the sample variable are:-

P(S)=[red,green,yellow,blue]=[4]

The favourable outcome is given as:-

F[s]=[1]

So the probability will be given by:-

P={1}÷{4}=0.25=0.25x 100=25\%

Hence the probability of landing on the 4 spinners divided into four equal parts will be P=25%

To know more about probability follow

https://brainly.com/question/24756209

#SPJ1

HELP ASAP
PLEASE HELP what is the measuring

Answers

Answer:

Step-by-step explanation:

Remark

No wonder you are posting this. You should never have to rely on a diagram to get an answer, and this diagram is particularly unreliable. What exactly does the peak mean? Does it tell you that the peak is both y and 20 degrees? I'm going to assume so. But it is a very shakey assumption.

Pick a triangle with 2 ys in it. The third angle hits one of the sides of the triangle. Every triangle has 180 degrees. The top angle is 20 degrees and all ys therefor are 20 degrees. So the third angle is y + y + x =180

The 2xs are in a quadrilateral near the top. All quadrilaterals have 360 degrees.  q and one of the angles are vertically opposite. q is then known.

Givens

y = 20

third angle = x

solution

triangles with 2 ys (isosceles)

y + y + x = 180

2y + x = 180

y = 20

2*20 + x = 180

40 + x = 180

x = 140

Quadrillateral

140 + 140 + 20 + vertically opposite q = 360

300 + vertically opposite q = 360

vertically opposite q = 360 - 300

vertically opposite q = 60

Answer

q = 60

What is the value of x in the triangle?
2xº
25°
25°

Answers

Answer:

130 degrees

Step-by-step explanation:

2x+25+25=180

2x=130

Answer:

65°

Step-by-step explanation:

25+25+2x= 180(sum of all the angles of a triangle)

2x= 180-50

x=130/2= 65

What percent is equivalent to the fraction 7/10

Enter your answer in the box.

Answers

Answer:

70%

Step-by-step explanation:

Two Steps to Convert a Fraction to a Percent

Use division to convert the fraction to a decimal:

7/10 = 7 ÷ 10 = 0.7

Multiply by 100 to get the percent value:

0.7 × 100 = 70%

Therefore the answer is 70%.

Given the following triangle:
В(2x+4)
A(2x-9)
Find the measure of angle A:
Find the measure of angle B:

Answers

Answer:

A=61, B=74

Step-by-step explanation:

Because all the angles in a triangle add up to 180:

(2x+4) + (2x-9) + x = 180

If you solve that for x, you will get that x=35. Now you just plug in x to the angles A and B.

What is the slope of the line that passes through the points ( 1 , − 1 ) (1,−1) and ( − 8 , − 7 ) (−8,−7)? Write your answer in simplest form.

Answers

[tex](\stackrel{x_1}{1}~,~\stackrel{y_1}{-1})\qquad (\stackrel{x_2}{-8}~,~\stackrel{y_2}{-7}) \\\\\\ \stackrel{slope}{m}\implies \cfrac{\stackrel{rise} {\stackrel{y_2}{-7}-\stackrel{y1}{(-1)}}}{\underset{run} {\underset{x_2}{-8}-\underset{x_1}{1}}}\implies \cfrac{-7+1}{-9}\implies \cfrac{-6}{-9}\implies \cfrac{-2}{-3}\implies \cfrac{2}{3}[/tex]

The slope of the line that passes through the points (1,−1) and (−8,−7) is m = 2/3.

What is the slope of a straight line?

Slope tells how vertical a line is. The more the slope is, the more the line is vertical. When slope is zero, the line is horizontal.

Its slope would be: [tex]m = \dfrac{y-q}{x-p}[/tex]

Slope of parallel lines are same. Slopes of perpendicular lines are negative reciprocal of each other.

If a line passes through two points, then its slope can be calculated with the following :

 [tex]m = \dfrac{y-q}{x-p}[/tex]

Plugging in our values, we get:

[tex]m = \dfrac{-7 + 1}{-8 - 1}[/tex]

Now, let's simplify our result:

[tex]m = \dfrac{-7 + 1}{-8 - 1}\\\\m = \dfrac{-6}{-9}[/tex]

m = 2/3

Therefore, the slope of the line that passes through the points (1,−1) and (−8,−7) is m = 2/3.

Learn more about slope here:

https://brainly.com/question/2503591

#SPJ2

A delivery truck drove 40 miles per hour. It took 2 hours to travel between two towns. What is the distance between the two​ towns

Answers

Answer:

80 miles

Step-by-step explanation:

40 miles per hour means you move 40 miles in one hour.

If it took 2 hours to travel between the two towns at 40 mph, multiply 40 by 2 and you get 80.

Hope this helps!

Answer:

your answer would be 80 miles

One angle of a right triangle measures 24°. What is the measure of the other acute angle?

Answers

Answer:

66 °

Step-by-step explanation:

Given:

One angle of a right triangle measures 24°.

To Find:

What is the measure of the other acute angle?

Solve:

Note:

Total angles in a triangle 180 degree

Right Triangle ⇒ 90 degree

Let "x" be the measure of the other acute angle

90° + 24° + x = 360°

114° + x = 180°

x = 66°

Kavinsky~

help ya girl out real quick!! 50 points for rewardd!

Answers

Answer:

Slope is 1

Step-by-step explanation:

Slope=RISE/RUN

You can see the slope is constant. To clarify we can do an example.

So rise can be 2

And run is also 2

Change in y is equal to change in x

so 2/2=1

Hope it helps

(4,0)(6,2)

Slope

m=y2-y1/x2-x1m=2-0/6-4m=2/2m=1

A spinner is numbered from 1 through 10 with each number equally likely to occur. What is the probability of obtaining a number less than 4 or greater than 8 in a single spin?

Answers

Answer:

i believe its 5/10 or 1/2

Step-by-step explanation:

Other Questions
I need help on this please Help me pls which one ahhh ??? Help will make brainlessly How many students are less than 69 inches in height19222517 Simplify 68.5/100 * 30. Pls ingore all of the questions below, but help me with the attached imageWhat is the only number that has the same number of letters as it's meaning? ...What number doesn't have its own Roman numeral? ...What is the only even prime number? ...What is the smallest perfect number? ...What is our current numerical system based on? ...Is Pi a rational or irrational number? write 3 ratios that are equivalent to 9/5 If r=1 and 0=5pie/3, what is the approximate arc length? Is deforestation a sudden or gradual change? which technological advancement made it easier for a small number of soldiers to repel a much larger force when defending Rennes during world war 1?O A. Machine gunsO B. Poison gasO C. TanksO D. AirplanesPLEASE HELP !!! In what season does the following date fall?Le dimanche 3 avril.O A. En t.O B. En automne.O C. Au printemps.O D. En hiver. Antonia likes to play chess as a hobby. To get an advantage over her potential chessopponents, she decides to run a simulation to see the most common opening moves in agame of chess. She sets her chess computer program to play 100 first moves in the gameof chess. Antonia concludes that of the 20 possible opening moves in chess, mostplayers will move the pawn piece in front of the queen forward two spaces..What are some potential problems with Antonia's simulation and her conclusion? Youneed to include 2 potential problems. Let f be the function given by f(x) = x^3 5x for what value in the closed interval [1,3] On July 4, 1776, the Declaration of Independence was adopted byA) the Continental CongressB) the Senate and the House of RepresentativesC) all American citizensD) Thomas Jefferson Quinton is an EMT. He arrives at the scene of an accident, and a person has been severely injured and clearly has several broken bones. At this point, what is the role of the EMT?A. to start setting the bonesB. to create a diagnosis and treatment planC. to refer the injured person to a specialistD. to keep the patient stable until they reach the hospital Multiply. - 5x(2xy - 3xy + 5) A. - 10xy + 15xy - 25x B. - 10xy - 15xy - 25x C. - 10xy + 15xy - 25x D. - 10xy - 15xy - 25x PLEASE HELP!!!What type of association does the following scatterplot plot representA-Positive linear associationB-Negative linear associationC-Positive nonlinear associationD-Negative nonlinear association What led to a substantial increase in population in the north in the 1800s? a) the west ran out of gold and people returned to the north in large numbers. b) white farmers disliked enslavement and decided to move to the free states in the north. c) a farming revolution led to more small farms being created in the north. d) an industrial revolution caused factories to be built and jobs to be created. HELPPP I DONT UNDERSTAND WILL GIVE BRIANLIESTCompared to the amount of carbon atoms on Earth, the amount of radioactive carbon atoms is infinitesimal. In this sentence, what is the best synonym for "infinitesimal?"A TinyB LargeC EqualD Dangerous Part b find the total number of pounds of cheese ordered. a. 11 pounds b. 13 pounds c. 15 pounds d. 17 pounds